Nested sequence of closed sets












4












$begingroup$


Is it true that every nested sequence of non-empty closed sets $(I_n)$ (one such that $I_{n+1} subset I_n$) has a non-empty intersection?










share|cite|improve this question











$endgroup$












  • $begingroup$
    Closed sets where? In what topological space?
    $endgroup$
    – Asaf Karagila
    Sep 10 '13 at 7:55






  • 3




    $begingroup$
    I added 'non-empty' to the question, because otherwise there is the trivial counter-example where $I_n =emptyset$ for large enough $n$.
    $endgroup$
    – Rhys
    Sep 10 '13 at 7:56


















4












$begingroup$


Is it true that every nested sequence of non-empty closed sets $(I_n)$ (one such that $I_{n+1} subset I_n$) has a non-empty intersection?










share|cite|improve this question











$endgroup$












  • $begingroup$
    Closed sets where? In what topological space?
    $endgroup$
    – Asaf Karagila
    Sep 10 '13 at 7:55






  • 3




    $begingroup$
    I added 'non-empty' to the question, because otherwise there is the trivial counter-example where $I_n =emptyset$ for large enough $n$.
    $endgroup$
    – Rhys
    Sep 10 '13 at 7:56
















4












4








4


2



$begingroup$


Is it true that every nested sequence of non-empty closed sets $(I_n)$ (one such that $I_{n+1} subset I_n$) has a non-empty intersection?










share|cite|improve this question











$endgroup$




Is it true that every nested sequence of non-empty closed sets $(I_n)$ (one such that $I_{n+1} subset I_n$) has a non-empty intersection?







real-analysis general-topology






share|cite|improve this question















share|cite|improve this question













share|cite|improve this question




share|cite|improve this question








edited Sep 10 '13 at 7:58







user94055

















asked Sep 10 '13 at 7:51









user94055user94055

2313




2313












  • $begingroup$
    Closed sets where? In what topological space?
    $endgroup$
    – Asaf Karagila
    Sep 10 '13 at 7:55






  • 3




    $begingroup$
    I added 'non-empty' to the question, because otherwise there is the trivial counter-example where $I_n =emptyset$ for large enough $n$.
    $endgroup$
    – Rhys
    Sep 10 '13 at 7:56




















  • $begingroup$
    Closed sets where? In what topological space?
    $endgroup$
    – Asaf Karagila
    Sep 10 '13 at 7:55






  • 3




    $begingroup$
    I added 'non-empty' to the question, because otherwise there is the trivial counter-example where $I_n =emptyset$ for large enough $n$.
    $endgroup$
    – Rhys
    Sep 10 '13 at 7:56


















$begingroup$
Closed sets where? In what topological space?
$endgroup$
– Asaf Karagila
Sep 10 '13 at 7:55




$begingroup$
Closed sets where? In what topological space?
$endgroup$
– Asaf Karagila
Sep 10 '13 at 7:55




3




3




$begingroup$
I added 'non-empty' to the question, because otherwise there is the trivial counter-example where $I_n =emptyset$ for large enough $n$.
$endgroup$
– Rhys
Sep 10 '13 at 7:56






$begingroup$
I added 'non-empty' to the question, because otherwise there is the trivial counter-example where $I_n =emptyset$ for large enough $n$.
$endgroup$
– Rhys
Sep 10 '13 at 7:56












3 Answers
3






active

oldest

votes


















4












$begingroup$

If $X$ is a compact space, this is true. $mathcal{F} = {I_alpha}_{alpha in I}$ is an arbitrary collection of nonempty closed sets such that any intersection of finitely many sets from $mathcal{F}$ is nonempty, then $bigcap_{alpha in I} I_alpha$ is nonempty.



To prove this, suppose $bigcap_{alpha in I} I_alpha$ is empty. Then ${X - I_alpha}$ is an open cover of $X$. Since $X$ is compact, there exists $alpha_1, ..., alpha_n$ such that $bigcup_{j = 1}^n (X - I_{alpha_j}) = X$. Then $bigcap_{j = 1}^n I_{alpha_j} = emptyset$. This contradicts the assumption that any finite intersection of elements of $mathcal{F}$ is nonempty.



For example (in your case) if $mathcal{F}$ consists of nonempty nested intervals, then $mathcal{F}$ has the finite intersection property and the above proof assured that the infinite intersection is nonempty.






share|cite|improve this answer









$endgroup$





















    5












    $begingroup$

    No: take $I_n=[n,to)={xinBbb R:xge n}$. If, however, some $I_m$ is compact, then $I_n$ is compact for all $nge m$, and the intersection will be non-empty.






    share|cite|improve this answer









    $endgroup$













    • $begingroup$
      How do you use compactness to prove that?
      $endgroup$
      – user94055
      Sep 10 '13 at 7:54






    • 2




      $begingroup$
      @user94055: For $nge m$ let $U_n=Bbb Rsetminus I_n$. Show that if $bigcap_{nge m}I_n=varnothing$, then ${U_n:nge m}$ is an open cover of $I_m$ with no finite subcover. You may find this post from Dan Ma’s Topology Blog helpful.
      $endgroup$
      – Brian M. Scott
      Sep 10 '13 at 7:57












    • $begingroup$
      @BrianM.Scott: Can i ask you to help me about this two questions 1, 2, Thanks.
      $endgroup$
      – M.Sina
      Sep 10 '13 at 8:08








    • 1




      $begingroup$
      @M.Sina: I did see your questions, but I’m probably going to need to refresh my memory a bit before answering. I’m about to head off to bed now, but I’ll take a look tomorrow.
      $endgroup$
      – Brian M. Scott
      Sep 10 '13 at 8:20






    • 1




      $begingroup$
      @BrianM.Scott: thanks. good night :).
      $endgroup$
      – M.Sina
      Sep 10 '13 at 8:27



















    2












    $begingroup$

    I'm studying intro to analysis and I got that a nested sequence of closed and nonempty subsets of $mathbb{R}$ do not necessarily have a nonempty infinite intersection:



    $textbf{Claim 1}$: The set $A_{n} = (-infty, -n]$ is closed, $nin mathbb{N}$.



    $textit{proof}$: Let $x$ be a limit point for $A_{n}Rightarrow$ if we had $x>-n,$ choose $epsilon = x+nRightarrow x-epsilon=-n<displaystyle x-frac{epsilon}{2}Rightarrow
    V_{epsilon/2}(x)cap A_{n}=emptyset Rightarrow $
    contradiction $Rightarrow xin A_{n}$.



    $textbf{Claim 2}$: $A_{n+1}subseteq A_{n}forall n$.



    $textit{Proof}$: $xin A_{n+1}Rightarrow xleq-n-1<-nRightarrow xin A_{n}$.



    $textbf{Claim 3}$: The sequence $A_{n}supset A_{n+1}, A_{n}=(-infty,-n] forall n$ has
    empty infinite intersection.



    $textit{Proof}: bigcap_{n=1}^{infty}A_{n}neq emptyset Rightarrow$ choose $xin
    bigcap_{n=1}^{infty}A_{n}Rightarrow xleq -nhspace{.5pc}forall nRightarrow -xgeq nhspace{.5pc}forall nRightarrow$
    contradiction ($mathbb{N}$ is not bounded above)



    $textbf{Conclusion}$: By claims 1,2, and 3, we have a nested sequence of closed sets with empty infinite intersection.



    $underline{Legend}$:




    1. $V_{epsilon}(x)=(x-epsilon,x+epsilon)$

    2. Infinite intersection of $A_{n}=bigcap_{n=1}^{infty}A_{n}$






    share|cite|improve this answer









    $endgroup$














      Your Answer








      StackExchange.ready(function() {
      var channelOptions = {
      tags: "".split(" "),
      id: "69"
      };
      initTagRenderer("".split(" "), "".split(" "), channelOptions);

      StackExchange.using("externalEditor", function() {
      // Have to fire editor after snippets, if snippets enabled
      if (StackExchange.settings.snippets.snippetsEnabled) {
      StackExchange.using("snippets", function() {
      createEditor();
      });
      }
      else {
      createEditor();
      }
      });

      function createEditor() {
      StackExchange.prepareEditor({
      heartbeatType: 'answer',
      autoActivateHeartbeat: false,
      convertImagesToLinks: true,
      noModals: true,
      showLowRepImageUploadWarning: true,
      reputationToPostImages: 10,
      bindNavPrevention: true,
      postfix: "",
      imageUploader: {
      brandingHtml: "Powered by u003ca class="icon-imgur-white" href="https://imgur.com/"u003eu003c/au003e",
      contentPolicyHtml: "User contributions licensed under u003ca href="https://creativecommons.org/licenses/by-sa/3.0/"u003ecc by-sa 3.0 with attribution requiredu003c/au003e u003ca href="https://stackoverflow.com/legal/content-policy"u003e(content policy)u003c/au003e",
      allowUrls: true
      },
      noCode: true, onDemand: true,
      discardSelector: ".discard-answer"
      ,immediatelyShowMarkdownHelp:true
      });


      }
      });














      draft saved

      draft discarded


















      StackExchange.ready(
      function () {
      StackExchange.openid.initPostLogin('.new-post-login', 'https%3a%2f%2fmath.stackexchange.com%2fquestions%2f489242%2fnested-sequence-of-closed-sets%23new-answer', 'question_page');
      }
      );

      Post as a guest















      Required, but never shown

























      3 Answers
      3






      active

      oldest

      votes








      3 Answers
      3






      active

      oldest

      votes









      active

      oldest

      votes






      active

      oldest

      votes









      4












      $begingroup$

      If $X$ is a compact space, this is true. $mathcal{F} = {I_alpha}_{alpha in I}$ is an arbitrary collection of nonempty closed sets such that any intersection of finitely many sets from $mathcal{F}$ is nonempty, then $bigcap_{alpha in I} I_alpha$ is nonempty.



      To prove this, suppose $bigcap_{alpha in I} I_alpha$ is empty. Then ${X - I_alpha}$ is an open cover of $X$. Since $X$ is compact, there exists $alpha_1, ..., alpha_n$ such that $bigcup_{j = 1}^n (X - I_{alpha_j}) = X$. Then $bigcap_{j = 1}^n I_{alpha_j} = emptyset$. This contradicts the assumption that any finite intersection of elements of $mathcal{F}$ is nonempty.



      For example (in your case) if $mathcal{F}$ consists of nonempty nested intervals, then $mathcal{F}$ has the finite intersection property and the above proof assured that the infinite intersection is nonempty.






      share|cite|improve this answer









      $endgroup$


















        4












        $begingroup$

        If $X$ is a compact space, this is true. $mathcal{F} = {I_alpha}_{alpha in I}$ is an arbitrary collection of nonempty closed sets such that any intersection of finitely many sets from $mathcal{F}$ is nonempty, then $bigcap_{alpha in I} I_alpha$ is nonempty.



        To prove this, suppose $bigcap_{alpha in I} I_alpha$ is empty. Then ${X - I_alpha}$ is an open cover of $X$. Since $X$ is compact, there exists $alpha_1, ..., alpha_n$ such that $bigcup_{j = 1}^n (X - I_{alpha_j}) = X$. Then $bigcap_{j = 1}^n I_{alpha_j} = emptyset$. This contradicts the assumption that any finite intersection of elements of $mathcal{F}$ is nonempty.



        For example (in your case) if $mathcal{F}$ consists of nonempty nested intervals, then $mathcal{F}$ has the finite intersection property and the above proof assured that the infinite intersection is nonempty.






        share|cite|improve this answer









        $endgroup$
















          4












          4








          4





          $begingroup$

          If $X$ is a compact space, this is true. $mathcal{F} = {I_alpha}_{alpha in I}$ is an arbitrary collection of nonempty closed sets such that any intersection of finitely many sets from $mathcal{F}$ is nonempty, then $bigcap_{alpha in I} I_alpha$ is nonempty.



          To prove this, suppose $bigcap_{alpha in I} I_alpha$ is empty. Then ${X - I_alpha}$ is an open cover of $X$. Since $X$ is compact, there exists $alpha_1, ..., alpha_n$ such that $bigcup_{j = 1}^n (X - I_{alpha_j}) = X$. Then $bigcap_{j = 1}^n I_{alpha_j} = emptyset$. This contradicts the assumption that any finite intersection of elements of $mathcal{F}$ is nonempty.



          For example (in your case) if $mathcal{F}$ consists of nonempty nested intervals, then $mathcal{F}$ has the finite intersection property and the above proof assured that the infinite intersection is nonempty.






          share|cite|improve this answer









          $endgroup$



          If $X$ is a compact space, this is true. $mathcal{F} = {I_alpha}_{alpha in I}$ is an arbitrary collection of nonempty closed sets such that any intersection of finitely many sets from $mathcal{F}$ is nonempty, then $bigcap_{alpha in I} I_alpha$ is nonempty.



          To prove this, suppose $bigcap_{alpha in I} I_alpha$ is empty. Then ${X - I_alpha}$ is an open cover of $X$. Since $X$ is compact, there exists $alpha_1, ..., alpha_n$ such that $bigcup_{j = 1}^n (X - I_{alpha_j}) = X$. Then $bigcap_{j = 1}^n I_{alpha_j} = emptyset$. This contradicts the assumption that any finite intersection of elements of $mathcal{F}$ is nonempty.



          For example (in your case) if $mathcal{F}$ consists of nonempty nested intervals, then $mathcal{F}$ has the finite intersection property and the above proof assured that the infinite intersection is nonempty.







          share|cite|improve this answer












          share|cite|improve this answer



          share|cite|improve this answer










          answered Sep 10 '13 at 8:10









          WilliamWilliam

          17.4k22256




          17.4k22256























              5












              $begingroup$

              No: take $I_n=[n,to)={xinBbb R:xge n}$. If, however, some $I_m$ is compact, then $I_n$ is compact for all $nge m$, and the intersection will be non-empty.






              share|cite|improve this answer









              $endgroup$













              • $begingroup$
                How do you use compactness to prove that?
                $endgroup$
                – user94055
                Sep 10 '13 at 7:54






              • 2




                $begingroup$
                @user94055: For $nge m$ let $U_n=Bbb Rsetminus I_n$. Show that if $bigcap_{nge m}I_n=varnothing$, then ${U_n:nge m}$ is an open cover of $I_m$ with no finite subcover. You may find this post from Dan Ma’s Topology Blog helpful.
                $endgroup$
                – Brian M. Scott
                Sep 10 '13 at 7:57












              • $begingroup$
                @BrianM.Scott: Can i ask you to help me about this two questions 1, 2, Thanks.
                $endgroup$
                – M.Sina
                Sep 10 '13 at 8:08








              • 1




                $begingroup$
                @M.Sina: I did see your questions, but I’m probably going to need to refresh my memory a bit before answering. I’m about to head off to bed now, but I’ll take a look tomorrow.
                $endgroup$
                – Brian M. Scott
                Sep 10 '13 at 8:20






              • 1




                $begingroup$
                @BrianM.Scott: thanks. good night :).
                $endgroup$
                – M.Sina
                Sep 10 '13 at 8:27
















              5












              $begingroup$

              No: take $I_n=[n,to)={xinBbb R:xge n}$. If, however, some $I_m$ is compact, then $I_n$ is compact for all $nge m$, and the intersection will be non-empty.






              share|cite|improve this answer









              $endgroup$













              • $begingroup$
                How do you use compactness to prove that?
                $endgroup$
                – user94055
                Sep 10 '13 at 7:54






              • 2




                $begingroup$
                @user94055: For $nge m$ let $U_n=Bbb Rsetminus I_n$. Show that if $bigcap_{nge m}I_n=varnothing$, then ${U_n:nge m}$ is an open cover of $I_m$ with no finite subcover. You may find this post from Dan Ma’s Topology Blog helpful.
                $endgroup$
                – Brian M. Scott
                Sep 10 '13 at 7:57












              • $begingroup$
                @BrianM.Scott: Can i ask you to help me about this two questions 1, 2, Thanks.
                $endgroup$
                – M.Sina
                Sep 10 '13 at 8:08








              • 1




                $begingroup$
                @M.Sina: I did see your questions, but I’m probably going to need to refresh my memory a bit before answering. I’m about to head off to bed now, but I’ll take a look tomorrow.
                $endgroup$
                – Brian M. Scott
                Sep 10 '13 at 8:20






              • 1




                $begingroup$
                @BrianM.Scott: thanks. good night :).
                $endgroup$
                – M.Sina
                Sep 10 '13 at 8:27














              5












              5








              5





              $begingroup$

              No: take $I_n=[n,to)={xinBbb R:xge n}$. If, however, some $I_m$ is compact, then $I_n$ is compact for all $nge m$, and the intersection will be non-empty.






              share|cite|improve this answer









              $endgroup$



              No: take $I_n=[n,to)={xinBbb R:xge n}$. If, however, some $I_m$ is compact, then $I_n$ is compact for all $nge m$, and the intersection will be non-empty.







              share|cite|improve this answer












              share|cite|improve this answer



              share|cite|improve this answer










              answered Sep 10 '13 at 7:52









              Brian M. ScottBrian M. Scott

              461k40518920




              461k40518920












              • $begingroup$
                How do you use compactness to prove that?
                $endgroup$
                – user94055
                Sep 10 '13 at 7:54






              • 2




                $begingroup$
                @user94055: For $nge m$ let $U_n=Bbb Rsetminus I_n$. Show that if $bigcap_{nge m}I_n=varnothing$, then ${U_n:nge m}$ is an open cover of $I_m$ with no finite subcover. You may find this post from Dan Ma’s Topology Blog helpful.
                $endgroup$
                – Brian M. Scott
                Sep 10 '13 at 7:57












              • $begingroup$
                @BrianM.Scott: Can i ask you to help me about this two questions 1, 2, Thanks.
                $endgroup$
                – M.Sina
                Sep 10 '13 at 8:08








              • 1




                $begingroup$
                @M.Sina: I did see your questions, but I’m probably going to need to refresh my memory a bit before answering. I’m about to head off to bed now, but I’ll take a look tomorrow.
                $endgroup$
                – Brian M. Scott
                Sep 10 '13 at 8:20






              • 1




                $begingroup$
                @BrianM.Scott: thanks. good night :).
                $endgroup$
                – M.Sina
                Sep 10 '13 at 8:27


















              • $begingroup$
                How do you use compactness to prove that?
                $endgroup$
                – user94055
                Sep 10 '13 at 7:54






              • 2




                $begingroup$
                @user94055: For $nge m$ let $U_n=Bbb Rsetminus I_n$. Show that if $bigcap_{nge m}I_n=varnothing$, then ${U_n:nge m}$ is an open cover of $I_m$ with no finite subcover. You may find this post from Dan Ma’s Topology Blog helpful.
                $endgroup$
                – Brian M. Scott
                Sep 10 '13 at 7:57












              • $begingroup$
                @BrianM.Scott: Can i ask you to help me about this two questions 1, 2, Thanks.
                $endgroup$
                – M.Sina
                Sep 10 '13 at 8:08








              • 1




                $begingroup$
                @M.Sina: I did see your questions, but I’m probably going to need to refresh my memory a bit before answering. I’m about to head off to bed now, but I’ll take a look tomorrow.
                $endgroup$
                – Brian M. Scott
                Sep 10 '13 at 8:20






              • 1




                $begingroup$
                @BrianM.Scott: thanks. good night :).
                $endgroup$
                – M.Sina
                Sep 10 '13 at 8:27
















              $begingroup$
              How do you use compactness to prove that?
              $endgroup$
              – user94055
              Sep 10 '13 at 7:54




              $begingroup$
              How do you use compactness to prove that?
              $endgroup$
              – user94055
              Sep 10 '13 at 7:54




              2




              2




              $begingroup$
              @user94055: For $nge m$ let $U_n=Bbb Rsetminus I_n$. Show that if $bigcap_{nge m}I_n=varnothing$, then ${U_n:nge m}$ is an open cover of $I_m$ with no finite subcover. You may find this post from Dan Ma’s Topology Blog helpful.
              $endgroup$
              – Brian M. Scott
              Sep 10 '13 at 7:57






              $begingroup$
              @user94055: For $nge m$ let $U_n=Bbb Rsetminus I_n$. Show that if $bigcap_{nge m}I_n=varnothing$, then ${U_n:nge m}$ is an open cover of $I_m$ with no finite subcover. You may find this post from Dan Ma’s Topology Blog helpful.
              $endgroup$
              – Brian M. Scott
              Sep 10 '13 at 7:57














              $begingroup$
              @BrianM.Scott: Can i ask you to help me about this two questions 1, 2, Thanks.
              $endgroup$
              – M.Sina
              Sep 10 '13 at 8:08






              $begingroup$
              @BrianM.Scott: Can i ask you to help me about this two questions 1, 2, Thanks.
              $endgroup$
              – M.Sina
              Sep 10 '13 at 8:08






              1




              1




              $begingroup$
              @M.Sina: I did see your questions, but I’m probably going to need to refresh my memory a bit before answering. I’m about to head off to bed now, but I’ll take a look tomorrow.
              $endgroup$
              – Brian M. Scott
              Sep 10 '13 at 8:20




              $begingroup$
              @M.Sina: I did see your questions, but I’m probably going to need to refresh my memory a bit before answering. I’m about to head off to bed now, but I’ll take a look tomorrow.
              $endgroup$
              – Brian M. Scott
              Sep 10 '13 at 8:20




              1




              1




              $begingroup$
              @BrianM.Scott: thanks. good night :).
              $endgroup$
              – M.Sina
              Sep 10 '13 at 8:27




              $begingroup$
              @BrianM.Scott: thanks. good night :).
              $endgroup$
              – M.Sina
              Sep 10 '13 at 8:27











              2












              $begingroup$

              I'm studying intro to analysis and I got that a nested sequence of closed and nonempty subsets of $mathbb{R}$ do not necessarily have a nonempty infinite intersection:



              $textbf{Claim 1}$: The set $A_{n} = (-infty, -n]$ is closed, $nin mathbb{N}$.



              $textit{proof}$: Let $x$ be a limit point for $A_{n}Rightarrow$ if we had $x>-n,$ choose $epsilon = x+nRightarrow x-epsilon=-n<displaystyle x-frac{epsilon}{2}Rightarrow
              V_{epsilon/2}(x)cap A_{n}=emptyset Rightarrow $
              contradiction $Rightarrow xin A_{n}$.



              $textbf{Claim 2}$: $A_{n+1}subseteq A_{n}forall n$.



              $textit{Proof}$: $xin A_{n+1}Rightarrow xleq-n-1<-nRightarrow xin A_{n}$.



              $textbf{Claim 3}$: The sequence $A_{n}supset A_{n+1}, A_{n}=(-infty,-n] forall n$ has
              empty infinite intersection.



              $textit{Proof}: bigcap_{n=1}^{infty}A_{n}neq emptyset Rightarrow$ choose $xin
              bigcap_{n=1}^{infty}A_{n}Rightarrow xleq -nhspace{.5pc}forall nRightarrow -xgeq nhspace{.5pc}forall nRightarrow$
              contradiction ($mathbb{N}$ is not bounded above)



              $textbf{Conclusion}$: By claims 1,2, and 3, we have a nested sequence of closed sets with empty infinite intersection.



              $underline{Legend}$:




              1. $V_{epsilon}(x)=(x-epsilon,x+epsilon)$

              2. Infinite intersection of $A_{n}=bigcap_{n=1}^{infty}A_{n}$






              share|cite|improve this answer









              $endgroup$


















                2












                $begingroup$

                I'm studying intro to analysis and I got that a nested sequence of closed and nonempty subsets of $mathbb{R}$ do not necessarily have a nonempty infinite intersection:



                $textbf{Claim 1}$: The set $A_{n} = (-infty, -n]$ is closed, $nin mathbb{N}$.



                $textit{proof}$: Let $x$ be a limit point for $A_{n}Rightarrow$ if we had $x>-n,$ choose $epsilon = x+nRightarrow x-epsilon=-n<displaystyle x-frac{epsilon}{2}Rightarrow
                V_{epsilon/2}(x)cap A_{n}=emptyset Rightarrow $
                contradiction $Rightarrow xin A_{n}$.



                $textbf{Claim 2}$: $A_{n+1}subseteq A_{n}forall n$.



                $textit{Proof}$: $xin A_{n+1}Rightarrow xleq-n-1<-nRightarrow xin A_{n}$.



                $textbf{Claim 3}$: The sequence $A_{n}supset A_{n+1}, A_{n}=(-infty,-n] forall n$ has
                empty infinite intersection.



                $textit{Proof}: bigcap_{n=1}^{infty}A_{n}neq emptyset Rightarrow$ choose $xin
                bigcap_{n=1}^{infty}A_{n}Rightarrow xleq -nhspace{.5pc}forall nRightarrow -xgeq nhspace{.5pc}forall nRightarrow$
                contradiction ($mathbb{N}$ is not bounded above)



                $textbf{Conclusion}$: By claims 1,2, and 3, we have a nested sequence of closed sets with empty infinite intersection.



                $underline{Legend}$:




                1. $V_{epsilon}(x)=(x-epsilon,x+epsilon)$

                2. Infinite intersection of $A_{n}=bigcap_{n=1}^{infty}A_{n}$






                share|cite|improve this answer









                $endgroup$
















                  2












                  2








                  2





                  $begingroup$

                  I'm studying intro to analysis and I got that a nested sequence of closed and nonempty subsets of $mathbb{R}$ do not necessarily have a nonempty infinite intersection:



                  $textbf{Claim 1}$: The set $A_{n} = (-infty, -n]$ is closed, $nin mathbb{N}$.



                  $textit{proof}$: Let $x$ be a limit point for $A_{n}Rightarrow$ if we had $x>-n,$ choose $epsilon = x+nRightarrow x-epsilon=-n<displaystyle x-frac{epsilon}{2}Rightarrow
                  V_{epsilon/2}(x)cap A_{n}=emptyset Rightarrow $
                  contradiction $Rightarrow xin A_{n}$.



                  $textbf{Claim 2}$: $A_{n+1}subseteq A_{n}forall n$.



                  $textit{Proof}$: $xin A_{n+1}Rightarrow xleq-n-1<-nRightarrow xin A_{n}$.



                  $textbf{Claim 3}$: The sequence $A_{n}supset A_{n+1}, A_{n}=(-infty,-n] forall n$ has
                  empty infinite intersection.



                  $textit{Proof}: bigcap_{n=1}^{infty}A_{n}neq emptyset Rightarrow$ choose $xin
                  bigcap_{n=1}^{infty}A_{n}Rightarrow xleq -nhspace{.5pc}forall nRightarrow -xgeq nhspace{.5pc}forall nRightarrow$
                  contradiction ($mathbb{N}$ is not bounded above)



                  $textbf{Conclusion}$: By claims 1,2, and 3, we have a nested sequence of closed sets with empty infinite intersection.



                  $underline{Legend}$:




                  1. $V_{epsilon}(x)=(x-epsilon,x+epsilon)$

                  2. Infinite intersection of $A_{n}=bigcap_{n=1}^{infty}A_{n}$






                  share|cite|improve this answer









                  $endgroup$



                  I'm studying intro to analysis and I got that a nested sequence of closed and nonempty subsets of $mathbb{R}$ do not necessarily have a nonempty infinite intersection:



                  $textbf{Claim 1}$: The set $A_{n} = (-infty, -n]$ is closed, $nin mathbb{N}$.



                  $textit{proof}$: Let $x$ be a limit point for $A_{n}Rightarrow$ if we had $x>-n,$ choose $epsilon = x+nRightarrow x-epsilon=-n<displaystyle x-frac{epsilon}{2}Rightarrow
                  V_{epsilon/2}(x)cap A_{n}=emptyset Rightarrow $
                  contradiction $Rightarrow xin A_{n}$.



                  $textbf{Claim 2}$: $A_{n+1}subseteq A_{n}forall n$.



                  $textit{Proof}$: $xin A_{n+1}Rightarrow xleq-n-1<-nRightarrow xin A_{n}$.



                  $textbf{Claim 3}$: The sequence $A_{n}supset A_{n+1}, A_{n}=(-infty,-n] forall n$ has
                  empty infinite intersection.



                  $textit{Proof}: bigcap_{n=1}^{infty}A_{n}neq emptyset Rightarrow$ choose $xin
                  bigcap_{n=1}^{infty}A_{n}Rightarrow xleq -nhspace{.5pc}forall nRightarrow -xgeq nhspace{.5pc}forall nRightarrow$
                  contradiction ($mathbb{N}$ is not bounded above)



                  $textbf{Conclusion}$: By claims 1,2, and 3, we have a nested sequence of closed sets with empty infinite intersection.



                  $underline{Legend}$:




                  1. $V_{epsilon}(x)=(x-epsilon,x+epsilon)$

                  2. Infinite intersection of $A_{n}=bigcap_{n=1}^{infty}A_{n}$







                  share|cite|improve this answer












                  share|cite|improve this answer



                  share|cite|improve this answer










                  answered Oct 25 '18 at 19:15









                  JayJay

                  296




                  296






























                      draft saved

                      draft discarded




















































                      Thanks for contributing an answer to Mathematics Stack Exchange!


                      • Please be sure to answer the question. Provide details and share your research!

                      But avoid



                      • Asking for help, clarification, or responding to other answers.

                      • Making statements based on opinion; back them up with references or personal experience.


                      Use MathJax to format equations. MathJax reference.


                      To learn more, see our tips on writing great answers.




                      draft saved


                      draft discarded














                      StackExchange.ready(
                      function () {
                      StackExchange.openid.initPostLogin('.new-post-login', 'https%3a%2f%2fmath.stackexchange.com%2fquestions%2f489242%2fnested-sequence-of-closed-sets%23new-answer', 'question_page');
                      }
                      );

                      Post as a guest















                      Required, but never shown





















































                      Required, but never shown














                      Required, but never shown












                      Required, but never shown







                      Required, but never shown

































                      Required, but never shown














                      Required, but never shown












                      Required, but never shown







                      Required, but never shown







                      Popular posts from this blog

                      Le Mesnil-Réaume

                      Ida-Boy-Ed-Garten

                      web3.py web3.isConnected() returns false always